Sei sulla pagina 1di 7

Math 320, Real Analysis I Solutions to Homework 3 Problems

Exercise 2.5.2. Associativity of Infinite Series:

(a) Prove that if an infinite series converges, then the associative property holds.
P
Proof. Let an be an infinite series that converges to L. This means the corresponding
sequence of partial sums, (sm ), converges to L. Now consider a regrouping of the terms

(a1 + a2 + · · · + an1 ) + (an1 +1 + · · · + an2 ) + (an2 +1 + · · · + an3 ) + · · ·


P
This yields the series bk , where b1 = a1 + a2 + · · · + an1 , b2 = an1 +1 + · · · + an2 ,
b3 = an2 +1 + · · · + an3P, and in general bk = (ank−1 +1 + · · · + ank ) for each k ∈ N. We
claim that the series bk converges to L as well. So consider its sequence of partial
sums, (t` ), where
t` = b 1 + b2 + · · · + b `
for each ` ∈ N. Now

t` = b1 + b 2 + · · · + b `
= (a1 + a2 + · · · + an1 ) + (an1 +1 + · · · + an2 ) + · · · + (an`−1 +1 + · · · + an` )
= a1 + a2 + · · · + an`
= sn` ,
P
where sn` is the n` -th partial sumPof the original series an . Therefore, the sequence,
(t` ), of partial sums P
of the series bk is a subsequence of the sequence, (sm ), of partial
sums of the series an , namely (t` ) = (sn` ). Hence, by Theorem 2.5.2, since the
sequence (sm ) → L, its subsequence
P is convergent with the same limit, (sn` ) → L. That
is, (t` ) → L, so we conclude bk = L, which says that the series obtained by regrouping
also converges to L.

(b) Compare this result to the example discussed at the end of Section 2.1 where infinite
addition was shown not to be associative. Why doesn’t our proof in (a) apply to this
example?
Solution: The example at the end of Section 2.1 refers to the series (−1)n = −1 +
P
1 + −1 + 1 + −1 + 1 + · · · . The first grouping mentioned is

(−1 + 1) + (−1 + 1) + (−1 + 1) + (−1 + 1) + · · · = 0 + 0 + 0 + 0 + · · · = 0,

whereas in the second grouping we have

−1 + (1 + −1) + (1 + −1) + (1 + −1) + · · · = −1 + 0 + 0 + 0 + 0 + · · · = −1.

That first grouping corresponds to the subsequence of partial sums (s2n ) = (s2 , s4 , s6 , s8 , . . . ) =
(0, 0, 0, 0, . . . ), which clearly converges to zero, while the second grouping corresponds
to the subsequence (s2n−1 ) = (s1 , s3 , s5 , s7 , . . . ) = (−1, −1, −1, −1, . . . ), which
P we see
converges to −1. Therefore, the sequence of partial sums (sm ) of the series (−1)n has
subsequences which converge to different limits. This implies the sequence (sm ) diverges,
as in Example 2.2.7.
In fact, this is the reason that our proof in (a) does not P applyn to this example. Since the
sequence of partial sums, (sm ), diverges, the series (−1) is not convergent. Hence
n
P
(−1) does not satisfy the hypothesis in Part (a), so our proof does not apply to it.
Math 320, Real Analysis I Solutions to Homework 3 Problems

Exercise 2.5.6. Let (an ) be a bounded sequence, and define the set

S = {x ∈ R : x < an for infinitely many terms an }.

Show that there exists a subsequence (ank ) converging to s = sup S.

Proof. Let (an ) be a bounded sequence. Hence, by definition, there is a real number M > 0
such that |an | ≤ M for all n ∈ N. Therefore, −M ≤ an ≤ M for all n. This implies
−M − 1 ∈ S, so S is nonempty. Furthermore, if x ∈ S, then there must exist a term an for
which x < an . Thus x < an ≤ M , so M is an upper bound for the set S = {x ∈ R : x <
an for infinitely many terms an }, so S is bounded above. From this, we conclude that the set
S has a supremum, s = sup S, by the Axiom of Completeness.
We first show that, for all ε > 0 and for all ` ∈ N, there are infinitely many terms ak ∈ Vε (s)
ε
with k ≥ `. First, there may only be finitely many an with an > s + , for if there were
2
ε
infinitely many of them, then s + ∈ S so that s 6= sup S. Therefore, there is some M ∈ N
2
ε
such that an ≤ s + < s + ε whenever n ≥ M , i.e., there are at most finitely many terms
2
from the sequence that are greater than or equal to s + ε. Then, by Lemma 1.3.7, there must
be some element x ∈ S such that s − ε < x. Since x ∈ S, there are infinitely many an such
that x < an . Therefore, as s − ε < x, there are infinitely many terms an for which s − ε < an .
Of these, only finitely many may be larger than s + ε and only finitely many have indices less
than `, so there must be infinitely many terms an of the sequence satisfying both an ∈ Vε (s)
and n ≥ `.
Now consider the interval (s − 1, s + 1). By our previous paragraph, there are infinitely many
terms an such that an ∈ (s − 1, s + 1) and n ≥ 1. Let n1 be the smallest index such that the
term an1 ∈ (s − 1, s + 1). Then we consider the smaller interval (s − 21 , s + 12 ), which must
contain infinitely many terms an such that n ≥ n1 + 1. Let n2 be the smallest such index for
which n2 ≥ n1 + 1 > n1 and an2 ∈ (s − 21 , s + 12 ). Continue in this way, selecting nk to be the
smallest index such that nk ≥ nk−1 + 1 > nk−1 and ank ∈ (s − k1 , s + k1 ), which exists since
there are still infinitely many an ’s with n ≥ nk−1 + 1 and an ∈ (s − k1 , s + k1 ) by the previous
paragraph. Then we have inductively constructed a subsequence (ank ) of our given sequence,
(an ), which we now claim converges to s.
Let ε > 0 be arbitrary. By the Archimedean Property, there is a natural number K such that
1 1 1
< ε. Suppose k ≥ K. Then ≤ < ε, and
K k K
 
1 1 1
ank ∈ s − , s + , so |ank − s| < < ε
k k k

whenever k ≥ K. Therefore, (ank ) → s.

Exercise 2.6.6. Equivalent Statements of the Completeness of the Real Numbers:

(a) Use the Nested Interval Property to give a proof of the Axiom of Completeness.

Proof. Suppose that A ⊆ R is nonempty and bounded above. We want to show that A
has a least upper bound, which is the conclusion of the Axiom of Completeness. Since
A is nonempty, there is some element a ∈ A. As A is bounded above, there is a real
Math 320, Real Analysis I Solutions to Homework 3 Problems

number b ∈ R such that a0 ≤ b for all a0 ∈ A. Now consider the interval I1 = [a, b],
which contains an element of A and an upper bound for A.
If a = b, then a is an upper bound for A and, if b0 is any other upper bound of A, then
a ≤ b0 (since a ∈ A), so a = sup A by definition of supremum. Hence, in this case,
sup A exists. From now on, suppose a 6= b, so a < b. We will inductively select a nested
sequence of nonempty closed intervals I1 ⊇ I2 ⊇ · · · ⊇ In ⊇ · · · , whose intersection must
then be nonempty by the Nested Interval Property, which we are assuming is true.
We have already produced the first closed interval I1 = [a, b]. We will explicitly describe
the selection of the closed subinterval I2 of I1 and inductively explain how to select
the closed subinterval In+1 of In following the same pattern. Consider the midpoint
a+b
m1 = of I1 . If m1 is an upper bound for A, take I2 = [a, m1 ]. If not, set
2
I2 = [m1 , b]. In the first case, a ∈ I2 implies that A ∩ I2 6= ∅ and the right hand endpoint
of I2 is an upper bound for A. In the second case, when m1 is not an upper bound for
A, there is some element a0 ∈ A such that m1 < a0 . Yet b is a upper bound of A, so
m1 < a0 ≤ b implies a0 ∈ I2 , so again A ∩ I2 6= ∅ and the right hand endpoint of I2 is an
upper bound for A. In either case, I2 = [a2 , b2 ] satisfies the following conditions:
i. I2 = [a2 , b2 ] is a nonempty closed subinterval of I1 ,
ii. the width of I2 is b2 − a2 = 12 [b − a],
iii. b2 is an upper bound for A, and
iv. there is some element a0 ∈ A ∩ I2 .
Now suppose we have the nested chain of closed intervals I1 ⊇ I2 ⊇ I3 ⊇ · · · ⊇ In , where
each Ik = [ak , bk ] satisfies
i. Ik = [ak , bk ] is a nonempty closed subinterval of Ik−1 ,
1 b−a
ii. the width of Ik is bk − ak = [bk−1 − ak−1 ] = k−1 ,
2 2
iii. bk is an upper bound for the set A, and
iv. A ∩ Ik 6= ∅
an + bn
for k = 2, 3, . . . , n. Consider In and its midpoint m = , so an < m < bn . If m is
2
an upper bound for the set A, let In+1 = [an , m]. Otherwise, take In+1 = [m, bn ]. Then,
writing In+1 = [an+1 , bn+1 ], we still have an+1 < bn+1 , bn+1 is an upper bound for the
set A, and A ∩ In+1 6= ∅ (by the same argument given in the construction of I2 ). By
induction, for each n ∈ N, there is a closed interval In = [an , bn ] with the properties (i)
through (iv) listed above. In particular, this is a nested sequence of nonempty closed
intervals, so the Nested Interval Property implies there must be an element

\
s∈ In .
n=1

We claim that s is the supremum of the set A. To prove this, we will first show that s
is an upper bound for A and then demonstrate that s must be the least upper bound
using Lemma 1.3.7.
Suppose that s were not an upper bound for A. Then, by definition, there must exist
some element a0 ∈ A such that a0 > s. Take ε = a0 − s > 0 and consider the real
b−a
number . By the Archimedean Property, there is a natural number M ∈ N such
ε
b−a b−a
that < M . Moreover, for all n ∈ N, n ≤ 2n−1 implies that < M ≤ 2M −1 ,
ε ε
Math 320, Real Analysis I Solutions to Homework 3 Problems

b−a b−a
so M −1
< ε. Then s ∈ IM implies that bM − s ≤ bM − aM = M −1 < ε = a0 − s, so
2 2
bM < a0 which contradicts that bM is an upper bound for A. Thus s is an upper bound
for A.
b−a
Now, let ε > 0 be arbitrary. Consider the real number . By the Archimedean
ε
b−a b−a
Property, there is a natural number K ∈ N such that < K ≤ 2K−1 , so K−1 < ε.
ε 2
b−a
Then s − aK ≤ bK − aK = K−1 < ε and IK = [aK , bK ] ∩ A 6= ∅ implies that there exists
2
a0 ∈ A such that a0 ≥ aK > s − ε, so s = sup A by Lemma 1.3.7.

(b) Use the Monotone Convergence Theorem to give a proof of the Nested Interval Property.

Proof. Let I1 ⊇ I2 ⊇ I3 ⊇ · · · ⊇ In ⊇ · · · be a nested sequence of nonempty closed


intervals, In = [an , bn ] for all n ∈ N. Now consider the sequence (an ) of the left
hand endpoints of the intervals In . Since the intervals are nested, for each n we have
[an+1 , bn+1 ] = In+1 ⊆ In = [an , bn ], so it follows that an+1 ≥ an and bn+1 ≤ bn for
all n ∈ N. Therefore, our sequence (an ) is monotone increasing. Furthermore, (an )
is bounded, since for all n we have a1 ≤ an ≤ bn ≤ b1 . Therefore, by the Monotone
Convergence Theorem, the sequence (an ) converges, so let α = lim an .
We claim that α ∈ ∞
T
n=1 In , which will imply that this intersection is nonempty, and
thus prove the Nested Interval Property. Thus we must show that α ∈ In = [an , bn ] for
all n, which means we must demonstrate that an ≤ α ≤ bn is always true. First, because
the intervals are nested, we see that every bm serves as an upper bound for the sequence
(an ), so by the Order Limit Theorem we may conclude that α = lim an ≤ bm for every
m. Moreover, α ≥ an for each n, for if α < aN for some N , then |an − α| = an − α ≥
aN − α > 0 for all n ≥ N since (an ) is increasing, which contradicts that α = lim an .
T∞ an ≤ α ≤ bn for every n, so α ∈ In = [an , bn ] for each n ∈ N. Therefore,
Therefore,
α ∈ n=1 In , so this intersection is not empty.

(c) Use the Bolzano-Weierstrass Theorem to prove the Nested Interval Property.

Proof. Let I1 ⊇ I2 ⊇ I3 ⊇ · · · ⊇ In ⊇ · · · be a nested sequence of nonempty closed


intervals, In = [an , bn ] for all n ∈ N. Now consider the sequence (an ) of the left
hand endpoints of the intervals In . Since the intervals are nested, for each n we have
[an+1 , bn+1 ] = In+1 ⊆ In = [an , bn ], so it follows that an+1 ≥ an and bn+1 ≤ bn for
all n ∈ N. Therefore, our sequence (an ) is monotone increasing. Furthermore, (an )
is bounded, since for all n we have a1 ≤ an ≤ bn ≤ b1 . Therefore, by the Bolzano-
Weierstrass Theorem, the sequence (an ) has a subsequence (ank ) which converges, so let
α = lim ank .
We claim that α ∈ ∞
T
n=1 In , which will imply that this intersection is nonempty, and
thus prove the Nested Interval Property. Thus we must show that α ∈ In = [an , bn ]
for all n, which means we must demonstrate that an ≤ α ≤ bn is always true. First,
because the intervals are nested, we see that every bm serves as an upper bound for the
sequence (an ), and hence also of the subsequence (ank ), so by the Order Limit Theorem
we conclude that α ≤ bm for every m. Moreover, as in Part (b), α ≥ ank for each k,
and we know ak ≤ ank since k ≤ nk , so it follows that α ≥ ak for all k ∈ T N. Therefore,
an ≤ α ≤ bn for every n, so α ∈ In = [an , bn ] for each n ∈ N. Thus, α ∈ ∞ n=1 In , so this
intersection is not empty.
Math 320, Real Analysis I Solutions to Homework 3 Problems

(d) Use the Cauchy Criterion to prove the Bolzano-Weierstrass Theorem.

Proof. Let (an ) be a bounded sequence. Thus there exists a real number M > 0 such
that |an | ≤ M for all n ∈ N. That is, −M ≤ an ≤ M for every term an of the sequence,
of which there are infinitely many. Thus, if we bisect the interval [−M, M ], at least one
of the subintervals [−M, 0] or [0, M ] must still contain infinitely many of the terms an of
our sequence. Let I1 be a half which contains infinitely many terms and select an1 to be
one of the terms of our sequence that is in the interval I1 . Notice that the width of the
interval I1 is M . Now divide I1 in half and let I2 be a half that contains infinitely many
of the terms an . Select an2 ∈ I2 such that n2 > n1 , which we can do since I2 contains
infinitely many an ’s and the requirement that n2 > n1 means we exclude only finitely
many of these from consideration (which leaves us with infinitely many left) from which
to select an2 . Then I2 has width M/2. Continuing in this fashion, for each k, starting our
current interval Ik that possesses infinitely many terms an from the sequence (including
ank that we select in it), divide it in half and let Ik+1 be a half that still contains
infinitely many terms. Select ank+1 ∈ Ik+1 so that nk+1 > nk , which we may do since
this requirement excludes at most nk of the infinitely many terms an in the subinterval
Ik+1 , which leaves infinitely many left from which to select ank+1 . In this way, we have
constructed a subsequence (ank ) of (an ) and a nested sequence I1 ⊇ I2 ⊇ · · · ⊇ Ik ⊇ · · ·
of subintervals of [−M, M ] such that the width of Ik is given by M/2k−1 for each k.
We claim that our subsequence (ank ) is Cauchy, and hence convergent. Let ε > 0 be
arbitrary. Consider the real number M/ε. By the Archimedean Property, there is a
natural number K ∈ N such that M/ε < K, and we know that K ≤ 2K−1 , so it follows
that M/2K−1 < ε. Let k, ` ≥ K and consider |ank − an` |. Now ank ∈ Ik ⊆ IK and
an` ∈ I` ⊆ IK , so the greatest the distance between ank and an` can be is the width of
IK , which is M/2K−1 , so
M
|ank − an` | ≤ K−1 < ε
2
whenever k, ` ≥ K. Therefore, by definition, our subsequence is a Cauchy sequence,
so it converges by the Cauchy Criterion. Therefore, the bounded sequence (an ) has a
convergent subsequence, which proves the Bolzano-Weierstrass Theorem.

P
Exercise 2.7.6. (a) Show
P that if xn converges absolutely, and the sequence (yn ) is bounded, then the sum
xn yn converges.

Proof. Let M > 0 be a real number such that |yn | ≤ M for all n ∈ N, which exists since
(yn ) is bounded. Let ε > 0 be arbitrary. Then ε/M > 0 as well, so there is a natural
number N ∈ N such that
ε
||xm+1 | + |xm+2 | + · · · + |xn || = |xm+1 | + |xm+2 | + · · · + |xn | <
M
P
whenever n > m P ≥ N by the Cauchy Criterion for Series applied to |xn |, which
converges since xn is absolutely convergent.
P
Now consider the series xn yn . We will show that it is absolutely convergent, and
thus converges by the Absolute Convergence Test. That is, we will show that the series
Math 320, Real Analysis I Solutions to Homework 3 Problems

P
|xn yn | converges, so let n > m ≥ N and consider

||xm+1 ym+1 | + |xm+2 ym+2 | + · · · + |xn yn || = |xm+1 | |ym+1 | + |xm+2 | |ym+2 | + · · · + |xn | |yn |
≤ |xm+1 | M + |xm+2 | M + · · · + |xn | M
= [|xm+1 | + |xm+2 | + · · · + |xn |] M
h ε i
< M = ε.
M
P P
Therefore, |xn yn | converges
P by the Cauchy Criterion for Series, so xn yn is abso-
lutely convergent. Thus xn yn converges by the Absolute Convergence Test.

(b) Find a counterexample


P that demonstrates that Part (a) does not always hold if the
convergence of xn is conditional.
P P (−1)n+1
Example: Consider xn = , which is the Alternating Harmonic Series. We
n
already know that this series converges, but
 that its convergence is conditional. Now let
(yn ) be the bounded sequence (−1) n+1 , which is bounded as |yn | = 1 ≤ 1 for all n.
(−1)n+1
  
P P n+1
 P1
Then the series xn yn = (−1) = , which is the Harmonic
n n
Series, which we
Pall know diverges. Hence the result of Part (a) need not hold if the
convergence of xn is conditional.
P
Exercise 2.7.9. Ratio Test: Given a series an with an 6= 0, the Ratio Test states that if (an ) satisfies

an+1
lim
= r < 1,
an

then the series converges absolutely.

(a1 ) Why is there a number r0 satisfying r < r0 < 1?


Solution: Since r < 1, the Density of Q in R implies that there is a rational number r0
such that r < r0 < 1.
r+1
Another solution to this question is found by taking r0 = , the average of r and 1,
2
which must be strictly between them since r < 1.
(a2 ) Show that there is a natural number N such that n ≥ N implies |an+1 | ≤ |an |r0 .
P
Proof. Suppose the sequence (an ) of terms of the series an satisfies

an+1
lim = r < 1.
an

Let r0 be a real number



0 0
such that r < r < 1. Then set ε = r − r, which is positive since
an+1
r < r0 . Since lim exists, for our ε > 0, there exists a natural number N ∈ N such
an
that
an+1
− r < ε = r0 − r


an
for all n ≥ N . Hence, whenever n ≥ N ,

an+1 0 0 0 0
an ∈ (r − [r − r], r + [r − r]) = (2r − r , r ),

Math 320, Real Analysis I Solutions to Homework 3 Problems


= |an+1 | and an 6= 0 implies |an | > 0 for
an+1 0
an+1
so < r when n ≥ N . Now
an an |an |
|an+1 |
all n. Therefore, when n ≥ N , we have < r0 , so |an+1 | < |an |r0 as we needed to
|an |
show.
P 0 n
(b) Why does |aN | (r ) necessarily converge?

an+1
Solution: By the Order Limit Theorem, 0 ≤ r = lim , so 0 ≤ r < r0 < 1, which
an

0 0
P 0 n 0 0
P 0 n that |r | = r < 1. Then (r ) is a geometric series with ratio r and |r | < 1, so
implies
(r ) converges P by Example
P 2.7.5. Finally, by the Algebraic Limit Theorem for Series,
the series |aN | (r0 )n = |aN |(r0 )n converges as well.
P
(c) Show that |an | converges.

P an+1
Proof. Let an be a series with an 6= 0 for all n such that lim = r < 1. By
an
Part (a), there is a real number r0 with r < r0 < 1 and natural number N ∈ N such
that |an+1 | ≤ |an |r0 whenever n ≥ N . We claim that for all k ∈ N, |aN +k | ≤ |aN |(r0 )k .
When k = 1, this follows from Part (a) as mentioned above. Now suppose it is true that
|aN +k | ≤ |aN |(r0 )k and consider
h i
|aN +(k+1) | ≤ |aN +k |r0 ≤ |aN |(r0 )k r0 = |aN |(r0 )k+1 .

Thus, by induction, |aN +k | ≤ |aN |(r0 )k for all k ∈ N.


P
Now consider the series |an |, which we need to show is convergent. First, we break
the infinite series into two parts, a finite initial portion and an infinite tail, as

X N
X ∞
X N
X ∞
X
|an | = |an | + |an | = |an | + |aN +k |.
n=1 n=1 n=N +1 n=1 k=1

|an | is finite, so ∞
P∞
Clearly the finite sum N
P P
n=1 P n=1 |an | converges if and only if k=1 |aN +k |
converges. To prove that |a
k N +k | converges, we employ the Comparison Test, as
0 k 0 k
P
0 ≤ |aN +k |P
≤ |aN |(r ) for all k ∈ N and k |aN |(r ) convergesP by Part (b) implies that
our series, ∞ |a
k=1 N +k |, converges as well. Therefore, the series ∞
n=1 |an | converges.

Potrebbero piacerti anche